Vous êtes sur la page 1sur 4

Physics 2049 Final Exam Solutions

1. A charge Q = 4.5 × 10−9 C is distributed uniformly on a string which is stretched between


x = 0 and x = 3.0 m. What is the electric field at x = 4.0 m? Recall that 0 = 8.85 ×
10−12 C2 N−1 m−2 .

y
What’s the field here?

0 3.0 4.0 x

Answer : 10 N/C
Solution: Let L = 3.0 m be the length of th string, and let d = 1.0 m be the distance between
the right end of the string and the point P at x = 4.0 m. Divide the string into segments of
length dx, each of which has a charge dQ = (Q/L)dx. The electric at point P due to one of
these segments at a distance x to the right of the origin is
kdQ k(Q/L)dx
dEx = = . (1)
(L + d − x)2 (L + d − x)2
To find the net electric field at P , we integrate this over the length of the string:
Z L  
k(Q/L)dx 1 1
Ex = 2
= k(Q/L) − . (2)
0 (L + d − x) d d+L
Substituting the numbers given, we find Ex = 10 N/C at point P .

2. Two concentric conducting spheres have radii a and b, with b > a; the region between the
spheres is a vacuum. What is the capacitance C of this system of conductors?
Answer : 4π0 (ab)/(b − a)
Solution: Let’s place a positive charge Q on the inner conductor, and charge −Q on the outer
conductor. The electric field is nonzero only in the region between the conductors; in that
region it points radially outward, with a magnitude E = Q/(4π0 r2 ), where r the distance
from the center (this is easily determined using Gauss’s Law). To find the potential we need
to integrate the electric field:
Z a
Va − V b = − ~ · d~s
E
b
a dr Z
Q
= −
4π0 b r2
 
Q 1 1
= −
4π0 a b
Q b−a
= . (3)
4π0 ab
The capacitance is C = Q/∆V , so C = 4π0 ab/(b − a).

1
3. The figure shows three circuits consisting of concentric circular arcs (either half- of quarter-
circles of radii r, 2r, and 3r) and radial lengths. The circuits carry the same current. Rank
them according to the magnitude of the magnetic field produced at the center of curvature
(the heavy dot), greatest first.

(a) (b) (c)

Answer : a, c, b
Solution: For simplicity let’s assume that the current is counter-clockwise in all three cases
(since we want the magnitude of the magnetic field, the direction of the current doesn’t
actually matter). The outer semicircle of radius 3r makes the same contribution in all three
cases—a magnetic field out of the page. The semicircular arc of radius r in (b) produces
a field into the page, thereby reducing the net magnetic field; in (a) and (c) the arcs still
produce fields out of the page. Therefore, (b) produces the smallest magnetic field at the
center of curvature. Comparing (a) and (c), we see that (a) has a semicircular arc of radius
r, while (c) has a quartercircle of radius r and a second quartercircle of radius 2r; since this
second quartercircle is farther from the center of curvature, it makes a smaller contribution
to the magnetic field there, so (c) has a smaller magnetic field at the center of curvature than
(a).

4. The region of space with x > 0 is permeated with a uniform magnetic field of magnitude B
pointing in the negative z direction (i.e. into the page in the figure). The field is zero in the
region with x < 0. A loop of wire bent into the shape of an equilateral triangle of side length
a is moving in the x-y plane with constant velocity v = vx̂, v > 0. The triangular loop is
initially in the zero field region oriented with one side parallel to the y axis and the opposite
vertex toward the region of magnetic field. What is the magnitude of the induced emf in
the wire and the direction of the induced current immediately before the loop is completely
immersed in the magnetic field.

a
v

Answer : Bva, counter-clockwise

2
Solution: The area within the triangle occupied by the field is the area of an equilateral
triangle of side l(t) which is increasing at the rate dA/dt = vl(t). Thus, by Faraday’s law,
the magnitude of the emf is E = dΦ/dt = BdA/dt = Bvl(t). Just before the triangle is
completely immersed in the field l(t) ≈ a, so E = Bva. Since the flux out of the page is
decreasing, Lenz’s law tells us the induced current must tend to increase the flux out of the
page, which means it is counter-clockwise.

5. You are given two lightbulbs, both designed to operate at 120 V. One is a 100 W bulb, and
the other is a 60 W bulb. The two bulbs are connected in series across a 120 V power outlet.
What is the total power dissipated by this circuit?
Answer : 38 W
Solution: We first need to determine the resistance of each bulb. The power dissipated is
given by P = V 2 /R, so that R = V 2 /P . Using V = 120 V, we see that the 60 W bulb has a
resistance of 240 Ω, and the 100 W bulb has a resistance of 144 Ω. When connected in series,
these bulbs have an effective resistance of 384 Ω, and when connected across a 120 V source,
they would dissipate a power of P = (120 V)2 /384 Ω = 38 W.

6. An electromagnetic wave is traveling in the positive x-direction with its electric field along
the y-axis (Ey ) and its magnetic field along the z-axis (Bz ). Which of the following equations
is correct?
Answer : ∂Bz /∂x = −µ0 0 ∂Ey /∂t
Solution: This equation can be derived from Maxwell’s equations (as was done in lecture).
Alternatively, you can just take the form of the electric and magnetic fields and substitute into
the equation to see that it works. In this case, Ey = E0 sin(kx−ωt), and Bz = (E0 /c) sin(kx−

ωt). Substitute into the equation above, and recall that ω = ck, with c = 1/ µ0 0 .

7. An erect candle is moved from a distance toward a convex mirror. Its image is
Answer : always virtual, erect, and smaller than the candle.
Solution: You can figure this out by simple ray tracing. Alternatively, use the spherical mirror
equation:
1 1 1
= + . (4)
f i p
Solving for i, we have
fp p|f |
i= =− , (5)
p−f p + |f |
where we have used the fact that for a convex mirror, f < 0. Now the object distance p is
always positive (by definition), so the equation above tells us that i < 0, corresponding to a
virtual image. The magnification of the object is given by

i |f |
m=− = , (6)
p p + |f |

which is positive but less than one, which means that the image is erect but smaller than the
object.

8. A camera with a lens of focal length 6.0 cm takes a picture of a man who is 1.4 m tall standing
11 m away. The height of the image is

3
Answer : 0.77 cm
Solution: Since the object is so far away, the image distance is approximately the focal length
of the lens, so that i = 6.0 cm. The absolute value of the magnification is the ratio of
the image height to the object height: |m| = h0 /h; however, it is also |m| = i/p, so that
h0 = h(i/p) = (6.0 cm)(1.4/11) = 0.77 cm.

9. A lens with a refractive index of 1.5 is coated with a material of refractive index 1.2 in order
to minimize reflection. If λ is the wavelength of the incident light in the air, what is the
thinnest possible such coating?

air
11111111111111111111
00000000000000000000
00000000000000000000
11111111111111111111
00000000000000000000
11111111111111111111
00000000000000000000
11111111111111111111 n=1.2
11111111111111111111
00000000000000000000
00000000000000000000
11111111111111111111
00000000000000000000
11111111111111111111
00000000000000000000
11111111111111111111
00000000000000000000
11111111111111111111
00000000000000000000
11111111111111111111
00000000000000000000
11111111111111111111
00000000000000000000
11111111111111111111
00000000000000000000
11111111111111111111
00000000000000000000
11111111111111111111
00000000000000000000
11111111111111111111
00000000000000000000
11111111111111111111
n=1.5
00000000000000000000
11111111111111111111
00000000000000000000
11111111111111111111
00000000000000000000
11111111111111111111
00000000000000000000
11111111111111111111

Answer : 0.208λ
Solution: The light reflected from both interfaces suffers a 180◦ phase shift upon reflection. If
we want to minimize reflection, we would like these two reflected rays to interfere destructively,
which means that the path length difference between the two rays must be a half integer times
the wavelength of the light in the coating; i.e.,

2t = (λ/ncoating )(m + 1/2), (7)

with m an integer. The thinnest possible such coating corresponds to m = 0, so that t =


λ/(4ncoating ) = λ/(4.8) = 0.208λ.

10. Light of wavelength 750 nm passes through a slit 10−4 cm wide. The light emerging from the
slit is incident upon a screen 20 cm away. How wide is the central maximum of the diffraction
pattern?
Answer : 45 cm
Solution: The condition for minima in the diffraction pattern is a sin θ = mλ, with m =
1, 2, . . . (but not m = 0). The first minima occurs at m = 1, so that sin θ = λ/a = 0.75,
or θ = 48.6◦ , which is certainly not a small angle. If we call the distance between the slit
and the screen L, and the distance of the first minimum from the central maximum y, then
tan θ = y/L, or y = L tan θ = 22.7 cm. The width of the central peak is twice this value, or
about 45 cm.

Vous aimerez peut-être aussi